This is a Flaw/Descriptive Weakening question.

Lindsey says several people claim that our company was unfair when it failed to give bonuses to the staff. She suggests that perhaps they recalled that the company had promised that if it increased its profits over last year’s, the staff would all get bonuses. However, the company's profit was much smaller this year than it was last year. And the conclusion is that clearly, then, the company acted fairly.

Did the company act fairly? Well, the company promised that if profits increased, staff would get bonuses. Profits did not increase, so that promise just falls away. Sufficient failed, rule disappears.  So what is clear is that the company did not contradict this promise. But that doesn't mean the company acted fairly. That's a much broader claim.

Let’s also look at why people are claiming that the company acted unfairly. Lindsey suggests that they are claiming this because of the promise above. Okay, if that's the case, then they do not have a good claim since the company did not contradict this promise. But if they are claiming that the company acted unfairly for some other reason, then Lindsey is now the one making a flawed argument. The fact is we actually don't know why they are claiming that the company acted unfairly. If you spotted this, you can go hunting for the right answer choice. If not, you can use the process of elimination.

Correct Answer Choice (B) says the argument infers (conclusion descriptor) that an opinion is false merely because (premise descriptor) one potential reason for that opinion has been undermined. Exactly. Lindsey does successfully undermine a potential reason in her premises, but we do not know if people are claiming the company was unfair for that specific reason. That's why Lindsey's inference that the people's opinion is false (that the company didn't act unfairly) is flawed.

Answer Choice (A) says the argument relies on the opinions of certain unnamed people without establishing that those people were well informed on the matter. Sure, people are unnamed here, but Lindsey is not relying on their opinion to make her argument. If anything, she is contesting their opinion. Establishing that those people were well informed on the matter would be important only if Lindsey were trying to make an appeal to those people's authority, like if those people were subject matter experts.

An argument guilty of (A) would look something like the following. The company promised that if profits increased, staff would get bonuses. However, some people told me that profits did not increase. Note how this is different from Lindsey stating that the company’s profit did not increase as a fact. In the former case, it was merely an opinion of some unnamed people, and so to take that opinion as establishing the fact of the matter, we need to know who they were. Like if the CFO told her this, then it's more likely to be true. But if it was just some random employee, then we might be skeptical.

Answer Choice (C) says the argument dismisses (conclusion descriptor) a claim on the basis of (premise descriptor) certain irrelevant attributes of the people who made the claim. While the argument does dismiss the claim that the company acted unfairly, its premise does not say that these people have certain irrelevant attributes.

Here is an example guilty of (C)'s description. "We should pay no attention to the people claiming that the company acted unfairly. This is obvious when you consider that they wore red hats." This would be dismissing a claim on the basis of irrelevant attributes of the people making the claim. That's a specific version of attacking the source of the argument.

Answer Choice (D) says the argument confuses the size of a quantity with the amount by which that quantity has increased. What does this even mean? Say the size of a quantity is a hundred units. But it was originally thirty units. So it increased by seventy units. So I am confusing the hundred units with the seventy-unit increase? How is that even relevant to Lindsey's argument?

I think (D) attempts to lure you in simply by mentioning "quantity" and "increase" and the stimulus said something about profits being larger or smaller.

(D) might be more relevant if we made some major modifications to the stimulus. The sufficient condition in the promise was straight up failed because the company’s profit this year being smaller than last year's means that the company's profits did not increase. But let's change that. The edited promise reads "If the company had any profits at all this year, the staff would get bonuses." And we'll keep the facts the same: the company's profit was much smaller this year than it was last year.

Still a bit of a stretch, but this makes (D) a bit more relevant. Under this modified stimulus, the sufficient condition is triggered by the facts because even if the company’s profits decreased this year, the company still has positive profits. The comparison to last year's profits isn't relevant. And if Lindsey argues that the sufficient condition wasn't met, then she may be confusing a comparative quantity (this year's smaller profit versus last year's larger profit) with an absolute quantity (that there was any positive profit at all).

Answer Choice (E) says the argument overlooks the possibility that a policy can be fair even when it is not generous. I guess Lindsey overlooked this, but so what? (E) would be relevant if people claimed the company was not generous and Lindsey responded that the company was fair. But no such confusion takes place.


12 comments

This is an RRE question.

Note that while the question stem says “justify,” it could just as well have said "would most help to explain" or “most help to resolve the paradox.” This is a classic RRE question where we have to explain a phenomenon.

The stimulus says it is widely known that the rescue squads serving high mountain areas with treacherous weather save the lives of many mountain climbers every year. This makes sense. For example, I am sure many lives are saved every year in the dangerous Alps by rescue squads. Then it says, however, that many experienced climbers believe that the rising annual toll of deaths and injuries among climbers in these regions can be significantly reduced only by completely abolishing the rescue squads.

So first we have this phenomenon of rising annual toll of deaths and injuries despite the rescue squads who save a lot of lives. But then experienced climbers are saying that deaths and injuries can be reduced only by completely abolishing the rescue squads. This seems paradoxical because we would think that we need to double or triple our rescue squads to save more climbers. Why do experienced climbers believe the opposite?

Answer Choice (A) says it is difficult to recruit and train members for the rescue squads. (A) might have been an explanation if we were asked to come up with any reason at all why experienced mountain climbers might say that we needed to get rid of rescue squads. Maybe because it is impossible to recruit or train members. So just give up on the whole project.

However, the question is not asking us to come up with any random reason why the experienced climbers believe rescue squads should be abolished. Rather, it is asking us why the experienced climbers believe doing so is the solution to the problem of rising deaths and injuries. (A) does not address this.

Be careful of these traps and make sure you explain the phenomenon in context. In this case, we have to explain the experienced climbers’ beliefs specifically in relation to rising deaths and injuries. They’re not just saying let’s abolish the rescue squad. They’re saying let’s abolish the rescue squad because it’ll bring down the death toll.

Answer Choice (B) says the recording of deaths and injuries tends to be more accurate in regions served by rescue squads. This makes sense. While rescue squads likely keep good records and have various standards and definitions as an organized group, regions that are not served by them may rely on inconsistent, ad hoc numbers from amateur climbers.

If the experienced climbers were only concerned about bringing down the annual reported numbers, (B) might have been an answer. Note, however, that we are trying to fix the reality of deaths and injuries, and not just trying to lower the reported numbers. We are not saying that if we do not see a problem, it does not exist.

While reported numbers might drop if you get rid of rescue squads, the actual deaths and injuries will not. (B) is not respecting the fact that these experienced climbers are proposing a solution to a real problem, and therefore fails to resolve the paradox.

Answer Choice (C) says that people who commonly take risks with their lives and health do not expect others to take those risks to save them. For brevity, let’s call this subset of people “risk takers.” Obviously, not all risk takers will take risks specifically by climbing mountains, and some of them might be skydivers, etc. But surely a subset of risk takers will take risks by climbing mountains, and per (C), they are therefore not expecting to be rescued.

If this is the case, why would abolishing rescue squads have an impact on deaths and injuries? If risk-taking mountain climbers get stuck in snow, they are happy to snowboard down the avalanche. They will not care that there are no rescue squads and are already behaving as if there were none.

(C) is something we see commonly in RRE questions, where some answer choices exacerbate the issue by blocking a potential explanation. A potential explanation could have been that those who get injured or killed are precisely the people who expect to be rescued, as we will see later in Answer Choice (E). (C) is not only talking about the wrong subject (we need to talk about people who are likely to get injured or killed, not risk takers), it is also saying they do not expect to be rescued. We need climbers to think that they are protected from trouble because they will be saved, and (C) is doing the opposite.

This leads us to Answer Choice (E), which is the version we want.

(E) claims that the lower the risk of climbing a mountain is perceived to be, the greater the number of less competent climbers who attempt to climb it. We first have to recognize that (E) is a comparative claim, and that the unstated part of this comparative is that the greater the risk is perceived to be, the fewer the number of less competent climbers.

In addition, we need to make two assumptions, both fairly reasonable and definitely more so than any assumption the other answers require. The first assumption is that when you remove the rescue squads, you increase not just the actual risk, but also the perceived risk. In other words, if I know that one mountain is protected by rescue squads and another is not, my perceived risk of the unprotected mountain goes up as a result.

The second assumption is that incompetent climbers are more likely to get injured or killed. This is also reasonable. The level of competence of mountain climbers is likely a normal distribution, where some are experts, most are in the middle, and some are incompetent. Who do you think will be more likely to get injured or killed? Of course, the less competent ones.

With these two assumptions, experienced climbers are saying that the incompetent climbers are the problem, and that rescue squads invite these incompetent climbers to get themselves hurt by lowering the perceived risk of climbing mountains. We therefore have to get rid of the rescue squads.

Answer Choice (D) says most of the people injured or killed while mountain climbing were not adequately prepared for the dangers they would face. (D) sounds similar to (E). The assumption we needed in (E) about how the incompetent are disproportionately injured or killed is made explicit in (D). If (D) is combined with (E), we might have an even better answer choice.

However, this is not a co-op game. We cannot merge (D) and (E) to create a better answer choice. We have to evaluate the answers independently. (E) is weak insofar as the assumption that incompetent climbers are more likely to get injured or die is weak. But, as we already discussed, that’s a fairly reasonable assumption.

However, (D) is very weak without (E). (D) just claims that people who were injured or killed were unprepared. That does not explain what will happen once we get rid of the rescue squads. Will the less prepared stop climbing mountains as a result? If you think so, then you are hopping over into the world of (E). (D) doesn’t say that and hence it does not resolve the paradox on its own.


11 comments

This is a Point at Issue/Disagreement question.

The question stem says the dialogue provides the most evidence that they disagree. Because of the words “most evidence,” I am getting the sense that the disagreement is not going to be textual, but rather inferred.

I found this question challenging, and if you did as well, I would recommend using an approach we use in comparative passages in Reading Comprehension. For these passages, I read Passage A and take a pass at the answers, then read Passage B and take another pass at the answers. I tend to not do this for Disagreement questions because they are shorter, but there is obviously a spectrum. On one end are the complicated comparative passages, and on the other end, maybe a Disagreement question where each speaker only says one sentence. Perhaps this question falls on the simpler side of the spectrum for you, but if not, I recommend siloing the information. So let's just read Paula’s claim first and take a pass at the answers.

Paula says earthlike planets seem to be the rule rather than the exception, so there are probably intelligent beings on other planets. Furthermore, we can expect our radio telescopes to detect signs of such beings. This is our main conclusion, but there is a huge jump here. Just because there are intelligent beings does not mean that we can detect them, let alone by radio telescopes.

She addresses these points in the following claims. Alien scientists would have basically the same understanding of math and physics that humans have. She does not tell us why they would, but if I accept this premise, I suppose the following subconclusion becomes a bit easier to swallow. She says the alien scientists would thus inevitably discover gravity, electromagnetism, and other fundamental physical phenomena and then develop technologies such as radio communication. Gravity and electromagnetism are consequences of our understanding of math and physics. So over time, I guess one can say alien scientists would discover these physical phenomena and develop technologies based on them. But ultimately, that aliens would develop radio communication is a subconclusion used to support the main conclusion that we can expect our radio telescopes to detect signs of intelligent beings.

Let’s now evaluate the answers. Paula has to either agree or disagree with each of these answer choices, i.e., she has to support it or anti-support it. Any answer choice that Paula has expressed no opinion on is unsupported and immediately wrong.

Answer Choice (A) says they disagree about whether it is probable that there are intelligent alien beings who have developed radio communication. Paula says there probably are intelligent alien beings, and that alien scientists would have developed radio communication. If we combine the two claims, we can say it is probable that there are intelligent alien beings who have developed radio communication. (A) is supported.

Answer Choice (B) says they disagree about whether it is probable that there are intelligent alien beings who have the same legal or political systems that humans do. Paula did not say anything about legal or political systems, so this is unsupported. That means we can eliminate this answer.

Answer Choice (C) says they disagree about whether our technology is influenced by our cognitive makeup. You might be tempted to say Paula disagrees with this. Maybe she believes alien scientists would have the same understanding of math and physics as us because there is something objective and absolute about them that does not depend on unique physiology or cognitive makeup. And our technology depends on math and physics, so it is not influenced by physiology or cognitive makeup.

But remember that Paula did not explain why alien scientists have the same understanding of math and physics as humans. She could believe this is so because aliens have the same physiology or cognitive makeup as humans. You cannot decide between those two possibilities because her statements are consistent with either reading. (C) is unsupported and therefore we can eliminate it.

Answer Choice (D) says they disagree about whether there is likely to be intelligent life on other planets. Paula explicitly says this, so (D) is supported.

Answer Choice (E) says they disagree about whether scientists should spend more time and money on the search for intelligent extraterrestrial life. Paula does not talk about whether the amount of investment into the search for extraterrestrial life is enough. So (E) is unsupported and eliminated.

Let’s now move onto Ashley’s claims. Under timed conditions, you would only evaluate (A) and (D) because you have ruled out (B), (C), and (E) based on Paula’s claims. But since we are reviewing, let’s look at all of them.

Ashley says Paula’s claim is like saying they, the aliens, would inevitably have the same legal or political systems that humans do. Do you detect the sarcasm here? She says our science, math, and technology are unique outgrowths of our physiology, cognitive makeup, and environment. And her conclusion is that using radio telescopes to search for intelligent life is a waste of time and money.

Notice that while Ashley is not committed to whether aliens exist, she is saying that even if they did exist, they would not have the same science, mathematics, and technology as us. Ours are unique, so this implies that aliens would have different ones, just as they would have different legal or political systems if they even had one. And this is why Ashley thinks using radio telescopes to search for intelligent life is a waste of time and money. She does not believe aliens have radio communication.

Answer Choice (A) is anti-supported. Ashley would absolutely disagree that there probably are alien beings who have radio communication.

Answer Choice (D) is unsupported. While Ashley would say radio-communicating intelligent alien beings probably do not exist, everything she says is consistent with her believing that there are or are not some intelligent beings on other planets.

Answer Choice (C) is supported. Ashley believes our technology is a unique outgrowth of our cognitive makeup.

Answer Choice (B) is anti-supported. Ashley is making fun of Paula by talking about aliens who have the same legal or political systems as humans. It is strongly implied that she thinks there are not any.

Answer Choice (E) is unsupported. (E) is talking about general investment into the search for intelligent life. Perhaps Ashley thinks that we should keep searching for aliens, but in some other way.

So from Ashley's perspective, we can get rid of (D) and (E). If you combine the results of both, the only answer choice remaining is the Correct Answer Choice (A). Paula agrees with this. Ashley disagrees with this.


9 comments

This is a Necessary Assumption question.

The stimulus says that a new screening test detects certain polyps at such an early stage that it is generally unclear whether the polyps are malignant. But the risk that a polyp might be malignant leads doctors, in most cases, to have such polyps surgically removed, which is a dangerous process. Yet some of those polyps turn out not to be malignant. All of this is the premise and we have the conclusion in the following sentence. Thus, the new screening test can prompt dangerous operations that are not actually medically necessary.

This is a classic example of a bridging Necessary Assumption question. The first part of the conclusion is fine. Because the new screening test is what leads doctors to have polyps surgically removed, “prompting” is weak enough of a claim to be supported by the premise. But look at the part of the conclusion that says these operations are “not medically necessary.” What do we know about what is or is not medically necessary? This is the bit that needs to be bridged. The starting point of the bridge is the fact that some polyps turn out not to be malignant, and the end of the bridge is that these had better not be medically necessary.

Correct Answer Choice (B) acts as this bridge. It says surgical removal of nonmalignant polyps detected by the new screening test is not always medically necessary. This has to be true. Imagine if (B) were false and surgical removal of nonmalignant polyps was always necessary. Then this argument completely fails. In other words, it must be true that removing nonmalignant polyps sometimes is not necessary. That is the bare minimum in order for the premise to have any chance of supporting the conclusion.

Answer Choice (A) says a surgical operation that is dangerous is ethically justified only for treating a medical condition that is more dangerous. You should lose interest as soon as you see “ethically justified” because the argument here is not one of ethics. It is about whether the operations are necessary or not. Even if we assume that the medical condition is not more dangerous than the operation itself, the operation would then not be ethically justified since we failed the necessary condition. But so what? Whether this is ethical is not what the conclusion is arguing for.

Answer Choice (C) says if the new screening test encourages medically unnecessary operations, then probably it either should not be used or its use should be modified. I lost interest in the first half of (C), because again, the researcher tries to argue that the test is not medically necessary. We are trying to build a bridge from her premises to that conclusion, but (C) is assuming that the conclusion is already proven and building a bridge from the conclusion to some new territory.

Answer Choice (D) says a polyp detected by the new screening test should be surgically removed if it is malignant. Of course, remove it if it is malignant. This argument is about polyps that we do not yet know to be malignant. (D) is just giving us obvious instructions.

Answer Choice (E) says the screening test is medically useful only when it detects a polyp that requires treatment. Remember how we did not care whether an operation was ethically justified in (A)? Similarly, in (E), we do not care whether a screening test is medically useful. In addition, the stimulus does not tell us which polyps require treatment. So (E) is laying out a necessary condition that itself requires a bridge (e.g., polyps at risk of malignancy require treatment) in order to even connect to the stimulus. And even if you fail this necessary condition, at best, you find out that the screening test was not medically useful.


1 comment

This is a Strengthening question.

The psychologist's argument uses scientific reasoning, so we have a phenomenon and a hypothesis. The stimulus describes some observations, which can easily sound like an experiment. If interpreted as an experiment, it's a pretty badly designed experiment. So thinking about what controls we need to improve this "experiment," in other words, contrasting this with what an ideal experiment would look like, will help you solve this question.

The stimulus says most people's blood pressure rises when they talk, but extroverted people experience milder surges when they speak than do introverted people, for whom speaking is more stressful. So we have kind of a classic setup in the premise with two different groups of people and two different results. And the conclusion, or the hypothesis, is that this suggests that blood pressure increases result from the psychological stress of communicating rather than from the physical exertion of speech production.

This is strange. What if physical exertion is something that does affect blood pressure? It sounds reasonable to say that this might be a competing explanation that we would want to examine. If the psychologist had said that it is stress that causes blood pressure to increase and not, for example, the color of the hat someone is wearing, then sure. It makes sense that we would not examine the color of people’s hats since this is unlikely to affect blood pressure.

But the conclusion is ruling out physical exertion while the premise did not give us reasons to do so. In fact, physical exertion is not mentioned at all. And that is why this is a badly designed "experiment." It is not really even an experiment, but rather just an observation, as it does not control for physical exertion or anything else, for that matter. Imagine how weak this argument would be if introverts happened to physically exert more when they spoke than extroverts. Now not only does stress vary with blood pressure, physical exertion also varies with blood pressure. How can you say it is the stress, and not the physical exertion, that is causing the surge in blood pressure?

If you really wanted to figure out if it is psychological stress that causes increases in blood pressure, you would collect a bunch of people and randomly split them into two groups. You would not care if they were introverts or extroverts because these distinctions only mattered since they were thought to be good proxies for the speakers’ stress levels. The conclusion wasn't that introversion caused an increase in blood pressure. It was that stress caused the increase. And because the groups are totally random, people who do a lot of physical exertion and people who do not would be spread out across the two groups as well. Now you have controlled for physical exertion.

And then you intervene in one of the groups. It does not matter which one because they are equalized at the moment. You intervene by stressing out one of the groups as they speak (maybe by inducing conversation on controversial topics), and whatever differential blood pressure you observe, you can be pretty sure it was due to your psychological intervention. That is how you design an ideal experiment.

So you have two options if you want to strengthen this crappy argument. One option is to control for physical exertion: say that the amount of physical exertion is the same for both introverts and extroverts. Another is to say that physical exertion simply does not matter for blood pressure, just as the color of somebody’s hat does not matter.

Correct Answer Choice (D) strongly suggests that physical exertion doesn't matter. On first blush, (D) might seem irrelevant because it does not mention introverts, extroverts, or blood pressure. This is why I generally try not to anticipate answers in Strengthening or Weakening questions, especially in scientific reasoning. LSAT writers can always think of things I cannot think of, and I might be digging myself into a hole by trying to anticipate the answer.

(D) says deaf people experience increased blood pressure when they sign, i.e., communicate, but no change when they move their hands for any other reasons. So (D) is suggesting that it is not the physical exertion that is causing blood pressure to increase. If it were, you would expect to see an increase in both situations.

Do you see how physical exertion, while not equalized, has now at least been somewhat accounted for? (D) does not control for differences in physical exertion between the two groups (for example, it could still be the case that introverts move their hands more than do extroverts) but it does strongly suggest that even if there were differences in physical exertion, it would not be relevant, like the color of your hat.

So what is causing this blood pressure increase? In one situation, they are communicating, and in the other, they are not. So (D) is in fact also hinting that the stress attendant with communication might be causing blood pressure to increase. So (D) is great. Even with (D), you can still poke holes in the argument, but this is okay. In Strengthening questions, you just need to make the argument a little bit better.

Answer Choice (C) says introverted people who do not have chronically high blood pressure often sense the rises in blood pressure that occur when they speak in conversation. So there is a subset of introverts who do not have chronic high blood pressure, and this subset can sense their blood pressures go up. Maybe they feel flushed, dizzy, etc. This doesn't help the argument. Imagine if this group was not so sensitive and could not sense this increase. This would not change the fact that the blood pressure for this group still went up.

In addition, how does this group of introverts relate to introverts in general? And what about introverts who do have chronically high blood pressure? Are they as sensitive? What about extroverts who do or do not have chronic high blood pressure? (C) is silent on all of these questions and completely useless.

Answer Choice (E) is a little better than (C) in that at minimum, it is a comparison. (C) just told us something about a random subset of introverts, but (E) says extroverts are more likely to have chronically high blood pressure than are introverts. On top of that, extroverts are also more likely to take medication to lower their blood pressure.

So (E) is actually kind of relevant, and if anything, it is revealing another hole in the reasoning. It is telling us that extroverts are potentially carrying in this baggage that could undermine the conclusion. If extroverts are taking medication to lower their blood pressure, the observed difference in blood pressure between the two groups could just have been the result of this medication. (E) is offering a competing explanation for why the extroverts’ blood pressure did not go up as much. And I rarely say this, but if this were a Weakening question, I think (E) would actually be a great answer choice.

Answer Choice (A) is kind of interesting in its relationship to (E). (A) says medications designed to lower blood pressure do not keep people who take them from experiencing blood pressure fluctuations when speaking. We saw how (E) weakened the argument by presenting a potential alternative hypothesis. (A) is addressing this very weakness by saying that the extroverts’ medications do not prevent blood pressure surges. Maybe they are designed to just lower ambient levels of blood pressure and do not work when your blood pressure surges.

I do not think (A) is very relevant without its relation to (E), because when we consider (A) alone, we do not even know which group is taking this medication. So where (A) might be relevant is precisely when it saps energy away from (E). But notice how we are now just back to where we were in the beginning. The argument is not any stronger.

Answer Choice (B) says that in general, the lower one's typical blood pressure, the more one's blood pressure will increase under stress. Fair enough, but it seems like (B) is ignoring the argument and just reiterating the conclusion that stress makes your blood pressure go up. This is a classic wrong answer choice in Strengthening questions. It just leaves our argument as good or bad as it ever was.

First, we do not know who has the lower baseline blood pressure since the stimulus just said extroverts experience a milder surge than introverts. And even if we say that the higher surges for introverts were due to a lower baseline and milder surges for extroverts were due to a higher baseline, we are still left with the original hole in the argument. We do not know if it is the stress or the physical exertion that causes surges in blood pressure.


12 comments

This is a Strengthening question.

Be careful not to mistake the question stem for a Most Strongly Supported question. In a Strengthening question, you are choosing an answer choice that adds to the support of the argument, while in an MSS question, you are taking the stimulus to be true and finding the answer choice that receives the most support.

Hopefully we got over that hurdle, but there are many more hurdles in the argument itself. You first have to understand how the “even if” is functioning in the stimulus as well as be able to take the contrapositive of the premise. However, this is not just an exercise in formal logic like an SA or a PSA question where you just map out the conditionals. In fact, this is actually more of a phenomenon-hypothesis question where the correct answer choice blocks an alternative hypothesis.

Let’s look at the stimulus. It says you should only buy a frying pan that has a manufacturer's warranty. That sounds like a command. And as if expecting an objection, the author says even if it requires paying a little bit more, I should still go for the warranty. But what if I am not the kind of person that would ever find the warranty form, fill it out, and mail it in? And as if the author had also anticipated this second objection, we are told that even if you would never bother seeking reimbursement should the pan not work, you still need to buy the pan with the warranty.

Do you see what the “even if” is doing? Imagine if the author had said that one should buy a frying pan that has a manufacturer's warranty, full stop. That full stop still completely anticipates any potential objections. So in a sense, the “even if” is not doing much. It is just reinforcing that you do not get out of this rule with these possible objections. If you are buying a frying pan, you'd better buy one with a manufacturer's warranty (buy pan → warranty).

And the premise to this conclusion is that manufacturers will not offer a warranty on a product if doing so means that they will need to reimburse many customers because the product did not work well or last long (reimburse many → no warranty). Let’s take the contrapositive of this. If a manufacturer does offer the warranty, then it is not the case that they need to reimburse many customers. (warranty → not reimburse many). And this is why you should only buy things with manufacturer's warranties. So the unstated assumption in this argument is that a manufacturer's warranty is an indicator of high quality, which means that not having to reimburse many customers is an indicator of high quality.

That is the unstated assumption, but let’s evaluate this assumption. Is it actually the case that not having to reimburse many customers is an indicator of high quality? This is where your phenomenon-hypothesis framework comes in. Let’s say All-Clad, an American manufacturer of frying pans, sells many frying pans but very rarely reimburses customers because of bad products. How do you explain this phenomenon?

One hypothesis might be that they make good products. It works well and lasts long, so not many customers seek reimbursement. And this is what the author presumes. But can you think of an alternative hypothesis? Maybe seeking reimbursement is a difficult process. You have to fill in a thousand different forms and call five different departments. Or maybe the particular market segment for frying pans is just too lazy, like me, to seek reimbursement. Both hypotheses would weaken the argument because now the fact that not very many people call in the warranties is no reflection on product quality.

Correct Answer Choice (A) says that most people who buy a frying pan with a manufacturer's warranty would seek reimbursement should the pan fail to work or last long. This wrecks the hypothesis above. (A) strengthens the argument by knocking out a couple of these competing hypotheses so that the product being high quality is more likely to be the hypothesis that explains the low reimbursements. (A) has identified precisely where the argument is vulnerable and patched it up. It has reaffirmed an assumption the argument already made.

Answer Choice (B) says that all of the frying pans currently on the market covered by warranty work at least as well at the time of purchase as any of the pans not covered by warranty. On first blush, you might think (B) strongly supports the conclusion because it seems to provide another reason to buy the warranty pans. But this is a pretty common trap the LSAT writers use in Strengthening questions, which is to come up with independent reasons to support the conclusion. Unlike (A), (B) does not address any of the premises or unstated assumptions and just independently supports the conclusion.

But if you examine (B) closely, you will see it does not even independently provide you with a reason to buy the pans with warranties. In order for (B) to independently support the conclusion, we need to assume that it is really bad if the non-warranty pans worked better than the warranty pans at the moment of purchase. If we make that assumption, then (B) helps by precluding that possibility. But we don't need to make that assumption. Would it actually be really bad if the non-warranty pans worked better than warranty pans at the moment of purchase? Frying pans are not disposable like plastic forks. Durability matters. So what I care about is whether the non-warranty pan can keep up its quality, because I know if the quality starts degrading on my warranty pan, I can return it. So maybe I care a little bit that non-warranty pans are better than the warranty pans at the time of purchase, but it is not nearly as important as it might initially have seemed.

(B) is analogous to trap answer choices in Weakening questions that look like they are contradicting the conclusion, but upon closer inspection, they do not. This is a Strengthening question so (B)'s trap is to give you something that looks like it is supporting the conclusion, but upon closer inspection, it does not.

Answer Choice (E) is similar to (B). (E) says most frying pans’ manufacturer’s warranties provide for full customer satisfaction. So (E) gives us yet another reason to buy a pan with a warranty. (E) might be a little bit better than (B) because it supports the conclusion more strongly, but (E) still shares the same flaw as (B), which is that it completely ignores the argument. (E) is not entirely irrelevant, just as (B) was not entirely irrelevant in that how good the pan is at the time of purchase surely matters some. But neither (E) nor (B) meaningfully engages with the argument.

Answer Choice (D) says the most expensive frying pans are the ones most likely to work well for many years. But this information is not helpful because we do not know if the most expensive frying pans have a warranty or not. The author only says that one should go for the warranty even if it requires paying more, and never says warranty pans actually do cost more or less.

Answer Choice (C) says the more a frying pan costs, the more likely it is to be covered by a manufacturer's warranty. I suppose (C) is better than (D) because it is at least correlating cost to warranty. But it still does not do much since the author already said to forget the costs. The author thinks a manufacturer's warranty is an indication of the product's quality, so harping on about costs is not going to affect the argument.


36 comments

This is a Weakening question.

The journalist says when judges do not maintain strict control over their courtrooms, lawyers often try to influence jury verdicts by using inflammatory language and by badgering witnesses. And these obstructive behaviors hinder the jury's efforts to reach a correct verdict. Therefore, whenever lawyers engage in such behavior, it is reasonable to doubt whether the verdict is correct.

Does the conclusion follow? We know obstructive behaviors hinder the jury's effort to reach a correct verdict. But hinder isn't the same thing as prevent. That assumes the hindrance is effective. It's possible that the hindrance prevents the jury from reaching a correct verdict but it's also possible that the hindrance is simply an annoyance, something that the jury can shrug off. The point is that to hinder is just that, to hinder. It's not as dispositive as the argument wants us to believe.

Answer Choice (A) says court proceedings overseen by judges who are strict in controlling lawyers' behaviors are known to result sometimes in incorrect verdicts. I am already not interested in (A) because the argument is contemplating a specific situation where judges do not maintain control over their courtrooms. (A) does not engage with this premise and tells us that in a different world where judges do control their courtrooms, juries also reach wrong verdicts.

To make (A) better, we can edit the conclusion to say that only when lawyers engage in such behavior is it reasonable to doubt the veracity of the verdict. This version of the conclusion is saying that when we have a reason to doubt a verdict, it must be because of obstructive behavior. And (A) would weaken by illustrating a counterexample: juries mess up even when lawyers do not engage in obstructive behavior.

You will see a very similar cookie-cutter wrong Answer Choice (B) in Question 24 of this section, which also tells you to forget the situation at hand and talks about a different world. Just as you find patterns that repeat in the stimulus and correct answers, you will find recurring patterns in the wrong answer choices as well.

Answer Choice (B) says lawyers tend to be less concerned than are judges about whether the outcomes of jury trials are just or not. This is a comparative claim and somewhat obvious. We have an adversarial system where lawyers are concerned about winning and judges are concerned about justice. At best, (B) explains why lawyers have to be kept in check because they are not so concerned about justice.

Answer Choice (C) says people who are influenced by inflammatory language are very unlikely to admit they were influenced by such language. Let’s give (C) a charitable reading and say that the people influenced by inflammatory language here are the jurors. But even with this reading, (C) does not weaken the argument. If jurors were influenced, it does not matter if they are humble or reflective enough to admit that they were. If anything, (C) just tells us that this hindrance might actually lead to reasonable doubt about the verdict.

Answer Choice (E) says the selection of jurors is based in part on an assessment of the likelihood that they are free from bias. This is true in reality. For example, in a corporation versus an individual case, you would want jurors who are not already biased against corporations. However, the argument is not about bias. (E) would have been better if it said the selection of jurors is based in part on an assessment of the likelihood that they are free from being influenced by inflammatory language.

But even then, (E) would not be a great answer choice because it is denying the premise that obstructive behaviors do hinder the jury's effort to some extent. And while you do sometimes see explicit contradictions to a premise in the LSAT, this is generally not how the test writers weaken arguments.

Correct Answer Choice (D) says obstructive courtroom behavior by a lawyer is seldom effective in cases where jurors are also presented with legitimate evidence. This is the only answer choice that addresses the gap between the premise and the conclusion.

Remember we said that just because obstructive behaviors hinder the jury's efforts, we cannot assume that this hindrance is effective. And (D) confirms this. It says there's this other factor, legitimate evidence, that renders obstructive behaviors ineffective.

You might object and say (D) requires an assumption that cases where jurors are presented with legitimate evidence are most cases, or at minimum, the cases where lawyers engage in obstructive behaviors. But this is not an unreasonable assumption. We have a functioning judicial system with robust rules of evidence, and in the vast majority of cases, there is legitimate evidence. You might think it is unfair to be expected to know this, but what you are expected to know is how to identify the assumption that (D) requires and compare it to assumptions required by the other answer choices. While (D) may not be the perfect answer choice, it still is the best answer choice.


5 comments

This is a PSA question.

Mateo says global warming has caused permafrost to melt under several arctic villages, forcing all of their inhabitants to relocate at great expense. Then he says that pollution from automobiles is a major contributor to global warming (another premise). Now he's given us a causal chain starting with automobiles and leading to pollution, then global warming, then melting permafrost, and finally, expensive relocation. Here comes the conclusion: the automotive industry should be required to help pay for the villagers' relocation. We have descriptive causal premises leading to a prescriptive conclusion.

In PSA questions, we have the premise (P) and the conclusion (C) and are generally looking for a P → C "rule." And the reason the question stem includes the word "principle" is that the P → C could be stated in terms more general than what you see in the answer choices. And there is a large degree of freedom in how general they will be.

For example, the correct answer could generalize a lot and say any "entity" whose "actions" have some negative consequence must pay for the cost of that consequence. You can take this general principle and apply it to this argument, but you can also apply it to a bunch of other arguments. The correct answer could generalize less and say if an "industrial product" has some negative downstream consequences, then the "manufacturer" must pay to cover the costs of those consequences. This one would apply to a smaller set of arguments than the more generalized principle of entities.

Most PSA questions lay out recurring traps. We find one such trap, that of starting the bridge at the conclusion in Answer Choice (E). It says an industry that contributes to global warming should be required to pay for resulting damage to specific communities only if it has a general obligation to help pay for all damage produced by global warming.

An industry being required to pay is the desired conclusion. So that concept cannot be in the sufficient condition. At best, this rule allows the stimulus to fail the necessary condition (say we had a premise that amounted to the auto industry not having a general obligation to help pay for all damage), therefore concluding the failure of the sufficient (auto industry should not be required to pay for damage to specific communities). That's not what we want. This answer choice is logically wrong.

(E) would have been correct if it said an industry that contributes to global warming should be required to help pay for resulting damage to specific communities, full stop.

Correct Answer Choice (D) says any industry manufacturing a product whose use contributes to costly damage for others should be liable for damages generated by that product's use. This is the P → C rule. The information in our premise triggers the sufficient conditions: any industry (automobile) manufacturing a product (cars) whose use contributes to costly damage (melting-permafrost-induced relocation) for others (arctic villagers). Hence, we can draw the necessary condition as the conclusion: the automotive industry should be liable for damages.

Answer Choice (A) says any industry has an obligation to pay for any damage that it should have known would result from its activities. (A) is pretty good if you strike out “it should have known.” But as it stands, that condition doesn't match the premise and so the sufficient condition in (A) does not trigger. While it might be common sense that cars generate pollution, the long causal chain in the stimulus makes it less reasonable to say that the automotive industry should have known about the results of their activities on arctic villages. (A) exhibits the recurring defect of starting the bridge at the wrong location.

Answer Choice (B) says manufacturers should be required to produce goods in a way that minimizes harm to people and the environment. (B) just tells the automotive industry to change the way it produces cars, but does not help the villagers get paid. This rule delivers the wrong results. Also a recurring defect.

Answer Choice (C) says when the use of a product causes damages, governments should not be required to pay for the damage unless those responsible for manufacturing the product are also required to pay. (C) is saying that if the government is required to pay, then it must be that manufacturing is required to pay. Or, contrapositive, if the manufacturer isn't required to pay, then neither is the government.

This is a strange principle to talk about when you are trying to get arctic villagers paid. Maybe (C) thinks Mateo’s next argument is to petition the government to compensate these villagers, and it is saying that if you want the government to pay, you must first establish that manufacturing has to pay. How bizarre.

This rule also exhibits the recurring defects of failing to match the premise and failing to match the conclusion.


2 comments

This is a Flaw/Descriptive Weakening question.

Lindsey says several people claim that our company was unfair when it failed to give bonuses to the staff. She suggests that perhaps they recalled that the company had promised that if it increased its profits over last year’s, the staff would all get bonuses. However, the company's profit was much smaller this year than it was last year. And the conclusion is that clearly, then, the company acted fairly.

Did the company act fairly? Well, the company promised that if profits increased, staff would get bonuses. Profits did not increase, so that promise just falls away. Sufficient failed, rule disappears.  So what is clear is that the company did not contradict this promise. But that doesn't mean the company acted fairly. That's a much broader claim.

Let’s also look at why people are claiming that the company acted unfairly. Lindsey suggests that they are claiming this because of the promise above. Okay, if that's the case, then they do not have a good claim since the company did not contradict this promise. But if they are claiming that the company acted unfairly for some other reason, then Lindsey is now the one making a flawed argument. The fact is we actually don't know why they are claiming that the company acted unfairly. If you spotted this, you can go hunting for the right answer choice. If not, you can use the process of elimination.

Correct Answer Choice (B) says the argument infers (conclusion descriptor) that an opinion is false merely because (premise descriptor) one potential reason for that opinion has been undermined. Exactly. Lindsey does successfully undermine a potential reason in her premises, but we do not know if people are claiming the company was unfair for that specific reason. That's why Lindsey's inference that the people's opinion is false (that the company didn't act unfairly) is flawed.

Answer Choice (A) says the argument relies on the opinions of certain unnamed people without establishing that those people were well informed on the matter. Sure, people are unnamed here, but Lindsey is not relying on their opinion to make her argument. If anything, she is contesting their opinion. Establishing that those people were well informed on the matter would be important only if Lindsey were trying to make an appeal to those people's authority, like if those people were subject matter experts.

An argument guilty of (A) would look something like the following. The company promised that if profits increased, staff would get bonuses. However, some people told me that profits did not increase. Note how this is different from Lindsey stating that the company’s profit did not increase as a fact. In the former case, it was merely an opinion of some unnamed people, and so to take that opinion as establishing the fact of the matter, we need to know who they were. Like if the CFO told her this, then it's more likely to be true. But if it was just some random employee, then we might be skeptical.

Answer Choice (C) says the argument dismisses (conclusion descriptor) a claim on the basis of (premise descriptor) certain irrelevant attributes of the people who made the claim. While the argument does dismiss the claim that the company acted unfairly, its premise does not say that these people have certain irrelevant attributes.

Here is an example guilty of (C)'s description. "We should pay no attention to the people claiming that the company acted unfairly. This is obvious when you consider that they wore red hats." This would be dismissing a claim on the basis of irrelevant attributes of the people making the claim. That's a specific version of attacking the source of the argument.

Answer Choice (D) says the argument confuses the size of a quantity with the amount by which that quantity has increased. What does this even mean? Say the size of a quantity is a hundred units. But it was originally thirty units. So it increased by seventy units. So I am confusing the hundred units with the seventy-unit increase? How is that even relevant to Lindsey's argument?

I think (D) attempts to lure you in simply by mentioning "quantity" and "increase" and the stimulus said something about profits being larger or smaller.

(D) might be more relevant if we made some major modifications to the stimulus. The sufficient condition in the promise was straight up failed because the company’s profit this year being smaller than last year's means that the company's profits did not increase. But let's change that. The edited promise reads "If the company had any profits at all this year, the staff would get bonuses." And we'll keep the facts the same: the company's profit was much smaller this year than it was last year.

Still a bit of a stretch, but this makes (D) a bit more relevant. Under this modified stimulus, the sufficient condition is triggered by the facts because even if the company’s profits decreased this year, the company still has positive profits. The comparison to last year's profits isn't relevant. And if Lindsey argues that the sufficient condition wasn't met, then she may be confusing a comparative quantity (this year's smaller profit versus last year's larger profit) with an absolute quantity (that there was any positive profit at all).

Answer Choice (E) says the argument overlooks the possibility that a policy can be fair even when it is not generous. I guess Lindsey overlooked this, but so what? (E) would be relevant if people claimed the company was not generous and Lindsey responded that the company was fair. But no such confusion takes place.


9 comments

This is an RRE question.

Note that while the question stem says “justify,” it could just as well have said "would most help to explain" or “most help to resolve the paradox.” This is a classic RRE question where we have to explain a phenomenon.

The stimulus says it is widely known that the rescue squads serving high mountain areas with treacherous weather save the lives of many mountain climbers every year. This makes sense. For example, I am sure many lives are saved every year in the dangerous Alps by rescue squads. Then it says, however, that many experienced climbers believe that the rising annual toll of deaths and injuries among climbers in these regions can be significantly reduced only by completely abolishing the rescue squads.

So first we have this phenomenon of rising annual toll of deaths and injuries despite the rescue squads who save a lot of lives. But then experienced climbers are saying that deaths and injuries can be reduced only by completely abolishing the rescue squads. This seems paradoxical because we would think that we need to double or triple our rescue squads to save more climbers. Why do experienced climbers believe the opposite?

Answer Choice (A) says it is difficult to recruit and train members for the rescue squads. (A) might have been an explanation if we were asked to come up with any reason at all why experienced mountain climbers might say that we needed to get rid of rescue squads. Maybe because it is impossible to recruit or train members. So just give up on the whole project.

However, the question is not asking us to come up with any random reason why the experienced climbers believe rescue squads should be abolished. Rather, it is asking us why the experienced climbers believe doing so is the solution to the problem of rising deaths and injuries. (A) does not address this.

Be careful of these traps and make sure you explain the phenomenon in context. In this case, we have to explain the experienced climbers’ beliefs specifically in relation to rising deaths and injuries. They’re not just saying let’s abolish the rescue squad. They’re saying let’s abolish the rescue squad because it’ll bring down the death toll.

Answer Choice (B) says the recording of deaths and injuries tends to be more accurate in regions served by rescue squads. This makes sense. While rescue squads likely keep good records and have various standards and definitions as an organized group, regions that are not served by them may rely on inconsistent, ad hoc numbers from amateur climbers.

If the experienced climbers were only concerned about bringing down the annual reported numbers, (B) might have been an answer. Note, however, that we are trying to fix the reality of deaths and injuries, and not just trying to lower the reported numbers. We are not saying that if we do not see a problem, it does not exist.

While reported numbers might drop if you get rid of rescue squads, the actual deaths and injuries will not. (B) is not respecting the fact that these experienced climbers are proposing a solution to a real problem, and therefore fails to resolve the paradox.

Answer Choice (C) says that people who commonly take risks with their lives and health do not expect others to take those risks to save them. For brevity, let’s call this subset of people “risk takers.” Obviously, not all risk takers will take risks specifically by climbing mountains, and some of them might be skydivers, etc. But surely a subset of risk takers will take risks by climbing mountains, and per (C), they are therefore not expecting to be rescued.

If this is the case, why would abolishing rescue squads have an impact on deaths and injuries? If risk-taking mountain climbers get stuck in snow, they are happy to snowboard down the avalanche. They will not care that there are no rescue squads and are already behaving as if there were none.

(C) is something we see commonly in RRE questions, where some answer choices exacerbate the issue by blocking a potential explanation. A potential explanation could have been that those who get injured or killed are precisely the people who expect to be rescued, as we will see later in Answer Choice (E). (C) is not only talking about the wrong subject (we need to talk about people who are likely to get injured or killed, not risk takers), it is also saying they do not expect to be rescued. We need climbers to think that they are protected from trouble because they will be saved, and (C) is doing the opposite.

This leads us to Answer Choice (E), which is the version we want.

(E) claims that the lower the risk of climbing a mountain is perceived to be, the greater the number of less competent climbers who attempt to climb it. We first have to recognize that (E) is a comparative claim, and that the unstated part of this comparative is that the greater the risk is perceived to be, the fewer the number of less competent climbers.

In addition, we need to make two assumptions, both fairly reasonable and definitely more so than any assumption the other answers require. The first assumption is that when you remove the rescue squads, you increase not just the actual risk, but also the perceived risk. In other words, if I know that one mountain is protected by rescue squads and another is not, my perceived risk of the unprotected mountain goes up as a result.

The second assumption is that incompetent climbers are more likely to get injured or killed. This is also reasonable. The level of competence of mountain climbers is likely a normal distribution, where some are experts, most are in the middle, and some are incompetent. Who do you think will be more likely to get injured or killed? Of course, the less competent ones.

With these two assumptions, experienced climbers are saying that the incompetent climbers are the problem, and that rescue squads invite these incompetent climbers to get themselves hurt by lowering the perceived risk of climbing mountains. We therefore have to get rid of the rescue squads.

Answer Choice (D) says most of the people injured or killed while mountain climbing were not adequately prepared for the dangers they would face. (D) sounds similar to (E). The assumption we needed in (E) about how the incompetent are disproportionately injured or killed is made explicit in (D). If (D) is combined with (E), we might have an even better answer choice.

However, this is not a co-op game. We cannot merge (D) and (E) to create a better answer choice. We have to evaluate the answers independently. (E) is weak insofar as the assumption that incompetent climbers are more likely to get injured or die is weak. But, as we already discussed, that’s a fairly reasonable assumption.

However, (D) is very weak without (E). (D) just claims that people who were injured or killed were unprepared. That does not explain what will happen once we get rid of the rescue squads. Will the less prepared stop climbing mountains as a result? If you think so, then you are hopping over into the world of (E). (D) doesn’t say that and hence it does not resolve the paradox on its own.


11 comments